81
Ph. D. Qualifying Exam Aug. 2017 Modern and Statistical Physics You can do all the problemsthe best 3 count towards your total score Problem 1: (40 points) Chemical potential of an ideal gas: (a) Show the chemical potential of an ideal gas in terms of the temperature T and the volume V can be expressed as: 0 ln ln P V cT cT T RT V sT const where is the chemical potential, P c is the specific heat at constant pressure, V c is the specific heat at constant volume, R is the gas constant, and 0 s is an entropy integration constant. [20 points] (b) Similarly, find in terms of T and P . In other words, show that the chemical potential at the fixed temperature T varies with pressure: 0 0 ln P RT P where 0 is the value of at the reference point 0 ( , ) PT . [20 points]

Ph. D. Qualifying Exam Jan. 2017 Modern and Statistical Physics

Embed Size (px)

Citation preview

Page 1: Ph. D. Qualifying Exam Jan. 2017 Modern and Statistical Physics

Ph. D. Qualifying Exam Aug. 2017

Modern and Statistical Physics

You can do all the problems—the best 3 count towards your total score

Problem 1: (40 points)

Chemical potential of an ideal gas:

(a) Show the chemical potential of an ideal gas in terms of the temperature

T and the volume V can be expressed as:

0

ln lnP Vc T c T T RT V sT const

where is the chemical potential, Pc is the specific heat at constant

pressure, Vc is the specific heat at constant volume, R is the gas

constant, and 0s is an entropy integration constant. [20 points]

(b) Similarly, find in terms of T and P . In other words, show that the

chemical potential at the fixed temperature T varies with pressure:

0

0

lnP

RTP

where 0 is the value of at the reference point 0( , )P T . [20 points]

Page 2: Ph. D. Qualifying Exam Jan. 2017 Modern and Statistical Physics

Problem 2: (40 points)

Black body radiation (Stefan-Boltzmann law):

(a) For black body radiation, the energy density /u U V depends on

only T . Derive the Stefan-Boltzmann law, 4e T , where 5 4

3 2

2.

4 15

ca kconst

h c

(the Stefan-Boltzmann constant), by

applying Bose-Einstein statistics. Here e is the energy flux

( / 4)( / )c U V , c is the wave velocity, 5 4 3 3(8 ) / (15 )a k h c , and k is

Boltzmann's constant. [20 points]

(b) Now obtain the above Stefan-Boltzmann law from classical

thermodynamics. In classical thermodynamics, it is found that

T V

U PT P

V T

For blackbody radiation, the energy density also depends on the

pressure (1/ 3)P u . Use the above classical thermodynamics

relation to obtain a differential equation for /u U V and its

solution, which is the Stefan-Boltzmann law. [20 points]

Problem 3: (40 points)

Assume the density of electrons in a white dwarf is 3010 /cm3. What are the

Fermi energy and average energy in eV of the degenerate electrons assuming

(a) the electrons are non-relativistic? [20 points]

(b) the electrons are ultra-relativistic? [20 points]

(in reality they are somewhere in between a and b)

Calculate numbers (in eV) using:

electron mass = 0.511 MeV/c2

1240hc eV nm

197c eV nm

Page 3: Ph. D. Qualifying Exam Jan. 2017 Modern and Statistical Physics

Problem 4: (40 points)

Show the energy levels, ordered in energy, for the states in neutral Helium

with 1n or 2 for the two electrons. Give the total J , L , and S plus the

degeneracy for each state, and explain why the energy ordering occurs. Give

a zeroth order estimate for the three primary energy levels (in eV) but do not

give numeric values for the shifts of those levels (just explain where the shifts

are coming from).

Problem 5: (40 points)

Each problem below has the same weight in points (~ 5.714 points each)

(a) Describe briefly how the following types of detectors work, indicating

the corresponding mechanisms that initiate the detection process and

the critical conditions for the proper operation of the detectors.

1. Geiger counter

2. Cerenkov counter

3. Bubble chamber

4. Cloud chamber

(b) Describe the dominant processes that give rise to high energy losses

(high attenuation) for the following particles (Note: there may be more

than one dominant process):

1. A 100 keV electron passing through matter.

2. A 100 MeV photon passing through matter.

3. A 100 GeV muon passing through matter.

Page 4: Ph. D. Qualifying Exam Jan. 2017 Modern and Statistical Physics

Ph. D. Qualifying Exam Jan. 2017

Modern and Statistical Physics

You can do all the problems—the best 3 count towards your total score

Problem 1: (40 points)

Show the energy levels, ordered in energy, for the states in neutral Helium

with 1n or 2 for the two electrons. Give the total J , L , and S plus the

degeneracy for each state, and explain why the energy ordering occurs. Give

a zeroth order estimate for the three primary energy levels (in eV) but do not

give numeric values for the shifts of those levels (just explain where the shifts

are coming from).

Problem 2: (40 points)

Assume the density of electrons in a white dwarf is 3010 /cm3. What are the

Fermi energy and average energy in eV of the degenerate electrons assuming

(a) the electrons are non-relativistic? [20 points]

(b) the electrons are ultra-relativistic? [20 points]

(in reality they are somewhere in between a and b)

Calculate numbers (in eV) using:

electron mass = 0.511 MeV/c2

1240hc eV nm

197c eV nm

Page 5: Ph. D. Qualifying Exam Jan. 2017 Modern and Statistical Physics

Problem 3: (40 points) A future colliding beam experiment is proposed to observe the process

e e HZ , where H is the Higgs boson, which subsequently decays (in a

majority of events) into a bottom quark-antiquark pair: H bb . In some

fraction of events, the Z boson can also decay into a bottom quark-antiquark

pair: Z bb . Let the e and e beams each have energy / 2E , and the

masses of the Higgs boson and the Z boson are HM and ZM . You may treat

the electron and the bottom quark as massless. The speed of light is 1c .

(a) What are the energies and the magnitudes of the 3-momenta of the

Higgs boson and the Z boson in the center-of-momentum frame?

[20 points]

(b) For bottom quark jets produced in such events, what is the maximum

possible energy? Do they come from Z decays or H decays?

[20 points]

[You do not need to plug in numbers, but the approximate relevant ones might

be: 300E GeV, 125HM GeV, 91ZM GeV.]

Problem 4: (40 points)

If a degree of freedom iq contributes a term 2iaq to the Hamiltonian, the

principle of equipartition says that its average contribution to the energy is 12kT .

(a) Calculate the rms deviation (or “energy fluctuations”) of 2iaq about its

average. [35 points] (note: rms = root mean square)

(b) Show that if the Hamiltonian contains N such terms 2

1

N

ii

aq

, with

average energy 12

E N kT , the rms deviation about this average is

then 2EN

showing that for a system with many degrees of

freedom, such as a macroscopic collection of molecules, the average

energy fluctuations about the mean are a very small fraction of the mean

energy. [5 points]

Page 6: Ph. D. Qualifying Exam Jan. 2017 Modern and Statistical Physics

Problem 5: (40 points)

(a) Using the Maxwell Relations (see below), evaluate an expression that

will enable you to calculate T

U

V

for a gas given its thermodynamic

equation of state , , 0f p T V . [25 points]

(b) For the Van der Waals equation of state for n moles of gas

2

2( )

anp V nb nRTV

derive an expression for T

U

V

. [15 points]

The Maxwell Relations (assuming the only work done is pdV work) can be

conveniently classified according to which pair of variable is chosen to be

independent.

1. S and V independent:

dQ TdS dU pdV dU TdS pdV

V

UT

S

,

S

Up

V

S V

T p

V S

--------- See Next Page ---------------

Page 7: Ph. D. Qualifying Exam Jan. 2017 Modern and Statistical Physics

2. S and p independent:

dH dU pdV Vdp TdS Vdp

p

HT

S

,

S

HV

p

pS

T V

p S

3. T and V independent:

dF dU TdS SdT pdV SdT

T

Fp

V

V

FS

T

V T

p S

T V

4. T and p independent:

dG dU TdS SdT pdV Vdp SdT Vdp

p

GS

T

T

GV

p

pT

S V

p T

Page 8: Ph. D. Qualifying Exam Jan. 2017 Modern and Statistical Physics

Ph. D. Qualifying Exam Aug. 2016

Modern and Statistical Physics

Do 4 out of 4 problems (the best 3 count)

Problem 1: (40 points)

Microwave Detector

A microwave detector is located at the shore of a lake at a height d above the

water level. As a radio star emitting monochromatic microwaves of

wavelength rises slowly above the horizon, the detector indicates

successive maxima and minima of signal intensity.

(a) What is the difference in phase between Ray 1 and Ray 2 arriving at the

detector (in terms of d , , and )? [20 points]

(b) At what angle above the horizon is the radio star when the first

maximum is received (in terms of d and )? [20 points]

d Ray 1

Microwaves

from Radio

Star

Ray 2

Detector

Lake

Page 9: Ph. D. Qualifying Exam Jan. 2017 Modern and Statistical Physics

Problem 2: (40 points)

Classical Thermodynamics

The equation of the sublimation and the vaporization curves for a particular

material are given by

ln 0.04 6 /P T (sublimation)

ln 0.03 4 /P T (vaporization)

where the pressure P is in atmospheres (atm), and the temperature T is in

Kelvin. (The following problem involves simple arithmetic calculations.)

For the following subsections, the subscripts 1, 2, and 3 denote solid, liquid,

and vapor phases, respectively.

(a) Find the temperature of the triple point TPT . [10 points]

(b) Show that the specific latent heats of vaporization 𝑙23 and sublimation

𝑙13 are 4R and 6R , respectively. Here R is the Gas constant.

(Assume that the specific volume in the vapor phase is much larger than

the specific volume in the liquid and solid phases). [20 points]

(c) Find the latent heat of fusion 𝑙12. [10 points]

Page 10: Ph. D. Qualifying Exam Jan. 2017 Modern and Statistical Physics

Problem 3: (40 points)

Boltzmann Gas Here we consider a non-interacting gas of N classical particles in three

dimensions. The gas is confined in a container of volume V . The energy of

the thi particle is given by 2 / (2 )i i m p .

(a) Calculate the canonical partition function

, ( )( )( , )

i i B

f

H k Tc NcZ T N e dh

q p

where H is the Hamiltonian of the system and d is the volume

element of the 2 -f dimensional phase-space spanned by the

components of the coordinates iq and momenta ip ( 1,..., )i N . What

is the number of spatial or momentum degrees of freedom, f , of the

gas? What is the value and meaning of the constant Nc ? Use the

thermal de Broglie wavelength, 2B Bh mk T , and the average

particle distance, 1/3( / )a V N , in your expression. [15 points]

(b) Calculate the free energy F from ( )cZ . F is the thermodynamic

potential associated with the canonical ensemble, i.e., [5 points] ( )ln ( , )c

BF k T Z T N

(c) Now, the particle number is not fixed anymore and we go over to the

grand canonical ensemble. Calculate the grand canonical partition

function by Laplace transformation

( ) ( )

0

( , ) ( , )BN

k Tgc c

N

Z T e Z T N

What is the meaning of ? [10 points]

(d) Show that the relation J F N for the thermodynamic potential

( , )J T (Planck-Massieu function) of the grand canonical ensemble

holds in the thermodynamic limit using the fact that the average particle

number in the grand canonical ensemble can be obtained as [10 points]

( )ln ( , )( / ( ))

gc

B

N Z Tk T

Hint: Use Stirling’s formula for large N : ! N NN N e

Do not forget to answer the short questions in (a) and (c)!

Page 11: Ph. D. Qualifying Exam Jan. 2017 Modern and Statistical Physics

Problem 4: (40 points)

Mandelstam Variables Consider the relativistic two-body scattering A B C D with

corresponding 4-vectors Ap , Bp , Cp , and Dp . The masses of the

corresponding objects are Am , Bm , Cm , and Dm . You can set the speed of

light as 1c if you would like. The Mandelstam variables are useful Lorentz

invariants:

2( )A Bs p p 2( )A Ct p p 2( )A Du p p

(a) Show that 2 2 2 2

A B C Ds t u m m m m . [10 points]

(b) Show that the total center of mass energy is given by s . [10 points]

(c) The LHC collides protons in the lab frame with an energy of 6.5 TeV

against other protons with an energy of 6.5 TeV (so that 13s TeV).

One of the things looked for is the production of new Z bosons. What

is the most massive Z that the LHC could directly observe?

[10 points]

(d) Why can the photon not decay? [10 points]

Hint 1: Prove that the mass of a decaying particle must exceed the

masses of the particles produced in its decay. In other words, if A

decays into B C , show that A B Cm m m .

Hint 2: Evaluate in the center of mass frame.

Page 12: Ph. D. Qualifying Exam Jan. 2017 Modern and Statistical Physics

Ph. D. Qualifying Exam Feb. 2016

Modern and Statistical Physics

Do 3 out of 5 problems

Problem 1: (40 points)

(a) The equation of state of a van der Waals gas is

(𝑃 +𝑎

𝑣2) (𝑣 − 𝑏) = 𝑅𝑇

where P is pressure, T is temperature, R is gas constant, v specific

volume, a and b are characteristic constants for a given gas.

Qualitatively explain the meaning of the terms 𝑎

𝑣2, and –b.

[10 points]

(b) A van der Waals gas undergoes an isothermal expansion from

specific volume v1 to specific volume v2. Calculate the change in

the specific Helmholtz function. [15 points]

(c) Calculate the change in the specific internal energy in terms of v1

and v2. [15 points]

Page 13: Ph. D. Qualifying Exam Jan. 2017 Modern and Statistical Physics

Problem 2: (40 points)

A negatively charged muon stops in aluminum, which has Z=13, where it is

captured and decays down to the 1S state.

(a) What is the energy of the gamma emitted when the muon drops from

the 2n to the 1n atomic state? Give your answer in terms of the

ground state (1S) energy of hydrogen. [20 points]

If the muon then undergoes the transition: Al e Al

(b) What normally conserved quantity is violated? [5 points]

(c) What is the total kinetic energy of the electron in terms of the masses

of the muon, electron, and aluminum? [15 points]

masses: muon 105 MeV, electron 0.5 MeV, Aluminum 25000 MeV

Problem 3: (40 points)

One-dimensional relativistic gas: Here we consider a non-interacting gas

of N relativistic particles in one dimension. The gas is confined in a

container of length L , i.e., the coordinate of each particle is limited to

0 iq L . The energy of the thi particle is given by ci ip .

(a) Calculate the single particle partition function 1( , )Z T L for given

energy E and particle number N . [12 points]

(b) Calculate the average energy 1E and the heat capacity 1C per particle

from 1( , )Z T L . [12 points]

(c) Calculate the Boltzmann entropy ( , )BS E N of all N particles.

Consider them as indistinguishable. [16 points]

Hint: Use Stirling’s formula for large N : ! 2 N NN NN e .

Page 14: Ph. D. Qualifying Exam Jan. 2017 Modern and Statistical Physics

Problem 4: (40 points)

A cylindrical resistor has a radius b , length L , and conductivity 1 . At the

center of the resistor is a defect consisting of a small sphere of radius a

inside which the conductivity is 2 . The input and output currents are

distributed uniformly across the flat ends of the resistor.

(a) What is the resistance of the resistor if 1 2 ? [5 points]

(b) Approximate the spherical defect as:

a cylinder of radius a and length a .

concentric with the cylinder of the physical resistor

and centered lengthwise on the center of the physical resistor.

What is the resistance of the resistor if 1 2 ? [25 points]

(c) Estimate the relative change in the resistance to first order in 1 2

if 1 2 . (Make any assumptions needed to simplify your method

of estimation). [10 points]

Page 15: Ph. D. Qualifying Exam Jan. 2017 Modern and Statistical Physics

Problem 5: (40 points)

Estimate the following quantities, indicating how you arrived at your

estimate (do not try to calculate exact numbers—make only estimates):

Choose only 4 of the following 7 quantities: [10 points each]

(a) the frequency of radiation used in a household microwave oven,

(b) the energy yield of a fission bomb with a 30 kg uranium core,

(c) the energy of impact of the earth with a meteorite 10 meters in

diameter,

(d) the temperature of the sun,

(e) the temperature of a 60-watt light bulb filament,

(f) the speed of sound in helium gas at STP,

(g) the total length of blood capillaries in the human body

You may find the following useful:

Speed of sound in a gas: 1/2)( Pv , where /P VC C

The speed of sound in air is about 330 m/sec

Stefan’s Law: 4

BTF , where 86 10

B W/m2·K4

Wein’s displacement Law: 3

max2.9 10T m·K

The average human has about 4 liters of blood

Typical energy release in a fission process is about 200 MeV

Earth’s orbit has radius of about 8 light minutes

The radius of the sun is about 2 light seconds

The earth’s orbital velocity is about 30 km/sec

The sun’s energy flux at the earth’s surface is about 1.3 kW/m2

Page 16: Ph. D. Qualifying Exam Jan. 2017 Modern and Statistical Physics

Ph. D. Qualifying Exam Sept. 2015

Modern and Statistical Physics

Do 3 out of 5 problems

Problem 1: (40 points)

A parallel, thin, monochromatic laser beam of wavelength falls on a

diffraction grating at normal incidence. The grating spacing is d .

(a) Derive the grating equation for normal incidence giving the relation

between d , , and . [10 points]

(b) Next rotate the grating through an angle 90 around an axis that is

parallel to the grating lines. Derive the grating equation giving the

relation between d , , , and . [20 points]

(c) How does the interference pattern produced on a viewing screen differ

between parts (a) and (b)? (intuitive guesses are allowed). [10 points]

Page 17: Ph. D. Qualifying Exam Jan. 2017 Modern and Statistical Physics

Problem 2: (40 points)

Classical Thermodynamics

(a) A van der Waals gas undergoes an isothermal expansion from specific

volume 1v to specific volume 2v . Calculate the change in the specific

Helmholtz function. [20 points]

Hint: The equation of state of a van der Waals gas is

2

aP v b RT

v. Here P is the pressure, v the specific volume, T is

the absolute temperature, and R is the gas constant, the constant a and

b are the characteristic constants of the gas.

(b) For the above problem, calculate the change in the specific internal

energy in terms of 1v and 2v . [20 points]

Problem 3: (40 points)

Statistical mechanics There is a system of two identical particles, which may occupy any of the

three energy levels 0 1 20, , 3 .

The system is in thermal equilibrium at temperature T , which means that the

system is a canonical ensemble. For each of the following cases, determine

the partition function and the energy and carefully enumerate the

configurations.

(a) The particles are Fermions. [10 points]

(b) The particles are Bosons. [10 points]

(c) The particles obey Boltzmann statistics and now they are

distinguishable. [10 points]

(d) Discuss the conditions under which Fermions or Bosons may be

treated as Boltzmann particles. [10 points]

Page 18: Ph. D. Qualifying Exam Jan. 2017 Modern and Statistical Physics

Problem 4: (40 points)

Electric Conductivity of Copper

The Drude model for a metal assumes that the conduction electrons can be

approximated by a gas of free electrons where the only important parameters

for the gas are n , the number density of electrons, and , the time between

collisions.

(a) Show that in this model the electrical conductivity of a metal can be

expressed as [24 points]

2ne

m

(b) Estimate the collision time for an electron in copper. The

resistivity of copper metal is 62 10 cm and the atomic density of

copper is 229 10 atoms/cm3. [16 points]

Page 19: Ph. D. Qualifying Exam Jan. 2017 Modern and Statistical Physics

Problem 5: (40 points)

The D0 meson , a combination of a charm quark and up anti-quark (𝑐), can

decay to K-mesons and Pi-mesons in several different channels. Three

possibilities are

𝐷0 → 𝐾− + 𝜋+ 𝐷0 → 𝜋− + 𝜋+ 𝐷0 → 𝐾+ + 𝜋−

Where a 𝐾+ meson is composed of 𝑢, a 𝐾− meson is composed of 𝑠, a 𝜋+

meson is composed of 𝑢, and a 𝜋− meson is composed of 𝑑.

(a) Draw the leading order Feynman diagrams for each of the above

decays. [10 points]

(b) Based on the quark flavors present in the initial state and the final

state, rank these decay channels from highest branching fraction to

lowest branching fraction. Explain your answer. [5 points]

(c) In a general two-body decay, particle A, at rest, decays into particles B

and C (𝐴 → 𝐵 + 𝐶). Find the energy and the magnitude of the

momentum of particles B and C. Express your answer in terms of the

particle masses (mA, mB, and mC) and the speed of light (c).

[25 points]

Page 20: Ph. D. Qualifying Exam Jan. 2017 Modern and Statistical Physics
Page 21: Ph. D. Qualifying Exam Jan. 2017 Modern and Statistical Physics
Page 22: Ph. D. Qualifying Exam Jan. 2017 Modern and Statistical Physics
Page 23: Ph. D. Qualifying Exam Jan. 2017 Modern and Statistical Physics
Page 24: Ph. D. Qualifying Exam Jan. 2017 Modern and Statistical Physics

Ph. D. Qualifying Exam Sept. 2014

Modern and Statistical Physics

Do 3 out of 5 problems

Problem 1: (40 points)

(a) A laser beam having a diameter D in air strikes a uniform flat piece of

glass of index of refraction n at an angle . What is the diameter of

the beam in the glass?

(b) A narrow beam of white light is incident in air at an angle on a

uniform flat sheet of glass of thickness L. The index of refraction for

red light is and for violet light it is . Determine the diameter of

the emerging beam assuming the incident beam is exceedingly

narrow.

Problem 2: (40 points)

Find the maximum total energy and maximum transverse momentum for the

initally at rest particle after the following elastic scatter reactions:

(a) a 10 GeV positron strikes an electron at rest

(b) a 10 GeV proton strikes an electron at rest

(c) a 10 GeV electron strikes a proton at rest

Note: mass electron = 0.5 MeV mass proton=1 GeV (for this problem)

Page 25: Ph. D. Qualifying Exam Jan. 2017 Modern and Statistical Physics

Problem 3: (40 points)

The system above is thermally isolated. It consists of two parts, a bucket

(A) containing a mixture of ice and water, and a body (B) with constant heat

capacity . The two parts are connected by a thermodynamic device which

can extract heat from part (A) and add heat to part (B).

The entire system is initially at temperature (the absolute temperature of

melting ice). The latent heat of melting of ice is per unit mass. Ignore all

effects due to changing volumes.

(a) How much heat must be removed from (A) to freeze an additional

mass of water? What is the entropy change of (A) in this process?

[10 points]

(b) In this process (B) absorbs the heat released by the thermodynamic

device, and its temperature increases to . Calculate the heat

absorbed and the entropy change of the body (B) in terms of

. [10 points]

(c) What is the minimum possible ? What is the minimum possible

? [20 points]

A B

Page 26: Ph. D. Qualifying Exam Jan. 2017 Modern and Statistical Physics

Problem 4: (40 points)

Calculate the radius of the largest atom which may be located

interstitially (without exercising stress) in a BCC lattice of a metallic system

whose atoms have a diameter of R.

Problem 5: (40 points)

(a) What are the two conditions for Fermi-Dirac statistics that the fermions

obey? [5x2 = 10 points]

(b) There is an assembly of N non-interacting fermions under the

conservation of particles and energy:

where Nj is the number of particles with single-particle energy εj; N and

U are fixed. There are gj quantum states at the jth energy level.

i. Derive the thermodynamic probability ωj of this assembly of

fermions for the jth energy level εj. [10 points]

ii. Using the above result, find the total number of microstates

corresponding to an allowable configuration ωFD. [5 points]

iii. Express and Derive the Fermi-Dirac distribution function for a

discrete energy level εj. Hint: Find the occupation number of each

energy level when the thermodynamic probability is a maximum,

i.e., the equilibrium macrostate Ni. Apply the method of Lagrange

multipliers for a function with two variables under the conservation

of particles and energy. Set one of the multipliers to be 1/kT and

the other to be μ/kT, where μ is the chemical potential.

[3 + 12 points]

Page 27: Ph. D. Qualifying Exam Jan. 2017 Modern and Statistical Physics

NIU Ph.D. Candidacy Examination Spring 2014 (2/22/2014)

Modern and Statistical Physics Do 3 out of 5 problems 1. [40 points] Classical thermodynamics, Entropy of the van der Waals gas.

a) Express the combined first and second law of thermodynamics. [5 points] b) Derive a Tds equation under the isochoric condition. Use specific heat under the isochoric

(constant volume) condition, cv. [20 points] c) Calculate the entropy for a van der Waals gas, using the above Tds equation. Here, the

equation of state of the van der Waals gas is P + av2

⎛⎝⎜

⎞⎠⎟ v − b( ) = RT . [15 points]

2. [40 points] Statistics of One-dimensional lattice vibration There is a one-dimensional lattice with lattice constant a as shown in figure below. An atom transits from a site to a nearest neighbor site every τ seconds. The probabilities of transiting to the right and left are p and q = 1 – p, respectively.

a) Calculate the average position x of the atom at the time t = Nτ , where N ≫1 . [20

points] b) Calculate the mean-square value x − x( )2 at the time t. [20 points]

3. [40 points] Special relativity, a rocketship travelling back home. A rocketship is traveling towards the Earth at 0.6c. It fires a projectile of mass M with a velocity of 0.8c in the rocketship's frame directly at the Earth. a) What is the energy of the projectile in the Earth's frame?

The rocketship is now traveling perpendicular to a line from the rocketship to the Earth at 0.6c and again fires a projectile at 0.8c in the rocketship's frame and at 90 degrees from the direction of the rocketship's motion (in the spaceship's frame). b) What is the energy of the projectile in the Earth's frame? c) What is the tangent of the angle of the projectile relative to the rocketship's motion in the

Earth’s frame?

You can ignore the effects of gravity as the rocketship is 2 light years away from the Earth.

Page 28: Ph. D. Qualifying Exam Jan. 2017 Modern and Statistical Physics

4. [40 points] Higgs boson. a) Show that the process γ → e+e− is forbidden in free space. [10 points] b) Deduce an expression for the energy of a γ from the decay of a Higgs boson: H →γγ

in terms of the Higgs boson mass (mH), the Higgs boson energy in the lab frame (E), the speed of the Higgs boson (β), and the emission angle (θ*) of one of the photons in the Higgs boson rest frame. [20 points]

c) The Higgs boson does not couple directly to photons. Draw the most important, lowest order Feynman diagram that illustrates the decay of the Higgs boson into two photons. Remember to label all the particles in your diagram. [10 points]

5. [40 points] Optics. A blade of grass standing ygrass tall is dA cm in front of a thin positive (convex) lens, A, having a focal length, fA. A thin negative (concave) lens, b, with a focal length of –fB is placed dB behind the first lens B. The blade of grass is illuminated by a monochromatic plane wave parallel to the optic axis of this combined lens system. a) Draw the ray diagram of imaging the above object by this combined lens system. [10

points] b) Where does the first lens, A, form the image of the blade of grass? Express it with the

above given parameters. What’s its magnification? [5 x 3 = 15 points] c) Where is the final image formed by this combined lens system? Express it with respect to

the location of the lens B. [10 points] d) What is the total magnification of this combined lens system? [5 points]

Page 29: Ph. D. Qualifying Exam Jan. 2017 Modern and Statistical Physics

1

NIU Ph.D. Candidacy Examination Fall 2013 (9/28/2013)

Modern and Statistical Physics Do 3 out of 5 problems 1. [40 points] Classical thermodynamics, Entropy change in mixing.

Two equal quantities of water, each of mass m and at temperatures T1 and T2 are adiabatically mixed together, the pressure remaining constant.

(a) Calculate the entropy change of the universe (thermodynamic surrounding) including cP is the specific heat capacity of the water at constant pressure. [25 points]

(b) Show that ΔS > 0 for any finite temperatures T1 and T2. [15 points] 2. [40 points] Fraunhofer diffraction of double slits. Consider an aperture consisting of two

long parallel openings of constant width b and separation d in an opaque screen illuminated by a coherent plane wave with a wavelength of λ. The length of the openings is L in the x direction, perpendicular to the paper, and long enough to ignore the diffraction effects of both ends. r0 lies in the yz-plane, and measures the distance from the geometric center of the grating to a point on the observation screen. The coordinates on the observation screen are (X, Y) and located far away to fulfill the Fraunhofer diffraction condition.

Fraunhofer diffraction geometry for a multiple slit aperture.

(a) What is the condition for Fraunhofer diffraction? Express it in a mathematical form. [5 x 2 = 10 points]

(b) Calculate the electric field distribution and intensity (irradiance) along Y-axis on the observation screen for a 2-slits (double slit) case. [20 points]

(c) Where along the Y-axis do bright fringes occur? Why? [5 points] (d) What makes a missing order? [5 points]

θ z

Screen d

b

r0

00

y Y

b

Page 30: Ph. D. Qualifying Exam Jan. 2017 Modern and Statistical Physics

2

3. [40 points] Fermi-Dirac gasses. We consider an ideal 3D Fermi gas comprising N noninteracting fermions, each of mass m, in a container of volume V held at temperature T.

(a) Express the Fermi function at T = 0. [5 points] (b) Find g ε( )dε , the number of quantum states whose energy lies in the range ε to ε + dε .

[10 points] (c) Find the average energy per fermion at absolute zero by making a direct calculation of

U(0)/N, where U(0) is energy at T = 0 for N fermions. Express this in terms of the Fermi energy, εF. [10 points]

(d) Similarly, calculate the average speed of a fermion gas particle at T = 0. Express this in terms of the Fermi velocity, vF. vF is defined by εF = 1 2( )mvF2 . [15 points]

4. [40 points] A particle with mass M and energy E, collides with another particle of mass M which is initially at rest. The result of the collision is two identical particles, each of mass m.

(a) Find a Lorentz transformation to the center-of-momentum frame, and find the energies of each of the two initial particles in that frame. [20 points]

(b) Find the maximum and minimum possible energies Emax and Emin of the two final state particle in the lab frame. [10 points]

(c) Find the energies of the two final state particles, if one of them is emitted at a right angle to the initial direction of the incident particle in the lab frame. [10 points]

5. [40 points] Statistical mechanics. The partition function of an Einstein solid is

Z = e−θ E 2T

1− e−θ E T ,

where θE is the Einstein temperature. Treat the crystalline lattice as an assembly of 3N distinguishable oscillators. (a) Calculate the Helmholtz function F. [10 points]

(b) Calculate the entropy S. [15 points] (c) Show that the entropy approaches zero as the temperature goes to absolute zero. [15

points]

Page 31: Ph. D. Qualifying Exam Jan. 2017 Modern and Statistical Physics

Ph. D. Qualifying Exam Feb. 2013

Modern and Statistical Physics Do 3 out of 5 problems Problem 1: (40 points) The specific Gibbs function of a gas is given by

APPP

RTg0

ln

where is a function of . Find expressions for:

(a) the equation of state [15 points]

(b) the specific entropy [10 points]

(c) the specific Helmhotz function [15 points]

Page 32: Ph. D. Qualifying Exam Jan. 2017 Modern and Statistical Physics

Problem 2: (40 points) The angle of deviation, , is defined as the angle between the direction of the incident beam and the direction of the beam transmitted by the prism.

(a) When the incident beam strikes the prism at an angle such that the angle of deviation is minimized, this minimum angle of deviation, , gives a simple relation for the index of refraction of the prism. This occurs when the angle of incidence for the 1st interface ( ) is equal to the angle of refraction for the 2nd interface ( ). Find the refractive index of refraction, , of the prism in terms of the angles and for this situation. [25 points]

(b) Find the angle of deviation, , for the general case (shown above) when it is not minimized. Find this angle in terms of the index of refraction, , of the prism, the incident angle of the 1st interface ( ), and . [15 points]

Page 33: Ph. D. Qualifying Exam Jan. 2017 Modern and Statistical Physics

Problem 3: (40 points)

(a) Derive an approximate formula for the strength of the magnetic field at the hydrogen nucleus produced by an electron in the first Bohr orbit (using the Bohr model of the atom). [20 points]

(b) Give a rough order of magnitude estimate of the strength of the magnetic field (in Teslas). Below are useful physical constants that you can use: [10 points] permeability of free space: N/A2 permittivity of free space: C2/Nm2 charge of the electron: C mass of the electron: kg

Jsec

(c) If the nuclear Bohr mageton is J/Tesla give an order of magnitude estimate of the hyperfine splitting (in eV) of spectral lines. [10 points]

Page 34: Ph. D. Qualifying Exam Jan. 2017 Modern and Statistical Physics

Problem 4: (40 points) Solve for the threshold energy for the following examples of the production of an electron-positron pair.

(a) Show the Feynman diagram for the process [5 points]

(b) What is the threshold energy for the photon (expressed in terms of electron masses) for the process in Part (a)? [5 points]

(c) What is the threshold energy for the photon (expressed in terms of

electron masses) for the process where is a free electron? [15 points]

(d) Why are the threshold energies in Parts (b) and (c) different? [5 points]

(e) In intergalactic space, protons can interact with the photons in the cosmic microwave background via

Outline the steps (but don't calculate) needed to obtain the proton threshold energy for this process. Assume the CMB photons all have the same energy ( degrees Kelvin). [10 points]

Page 35: Ph. D. Qualifying Exam Jan. 2017 Modern and Statistical Physics

Problem 5: (40 points) Measurements of the entropy of a certain paramagnetic salt, as a function of the temperature (in degrees Kelvin) and of the magnetic field (in gauss), have led to the following table (in units of per mole):

It is proposed to use this salt to produce very low temperatures by adiabatic demagnetization. The sample can be pre-cooled to 0.8 Kelvin by pumping on liquid He. The biggest available field is 10,000 gauss. What is the lowest temperature which can be reached? Each step in your reasoning should be explained very carefully (this is an essay question: numerical computations are not necessary)

Page 36: Ph. D. Qualifying Exam Jan. 2017 Modern and Statistical Physics

Ph. D. Qualifying Exam Sept. 2012

Modern and Statistical Physics Do 3 out of 5 problems Problem 1: (40 points) 1. Classical thermodynamics, Gay-Lussac-Joule experiment.

(a) For any reversible process, calculate the dependence of the specific internal energy on volume, !"

!" !, from the First law of

Thermodynamics. Here, the internal energy 𝑢 = 𝑢 𝑣,𝑇 . Here u and v are defined as u = U/n and v = V/n, where U, V, and n are the internal energy, volume, and a number of kilomoles. (hint: utilize the condition of an exact differential.) [25 points]

(b) For an ideal gas, find the dependence of 𝑢 on 𝑣 by calculating !"!" !

.

[15 points] Problem 2: (40 points) In a two-slit Young interference, the aperture-to-screen distance is 𝐿 and the wavelength is 𝜆!"#.

(a) What slit separation, 𝑎, is required to produce a fringe spacing of ∆𝑦 at the screen? [15 points]

(b) Assume a thin glass plate of thickness 𝑑 and index of refraction 𝑛 is placed over one of the slits (see figure below). The glass plate causes the entire fringe pattern to laterally shift up or down on the screen. Calculate the lateral fringe displacement. [25 points]

Screen

Glass plate

Page 37: Ph. D. Qualifying Exam Jan. 2017 Modern and Statistical Physics

Problem 3: (40 points) A 𝜋 meson of mass 𝑚! decays at rest into a muon (mass 𝑚!) and a neutrino of negligible mass. Express your answers below in terms of the masses 𝑚! and 𝑚!.

(a) What is the momentum of the muon? [15 points]

(b) What is the kinetic energy 𝐾! of the muon? [15 points]

(c) The muon decays, too. Let 𝜏! be the mean lifetime of the muon in its rest frame. What is the mean distance traveled by the muon in the rest frame of the 𝜋 meson? [10 points]

Problem 4: (40 points) A beam of either positively or negatively charged muons is stopped in a piece of Pb. Answer the following questions separately for both a positive muon and a negative muon. Hint: consider atom formation and the energy levels of a hydrogen-like atom.

(a) Where is the most likely place for the muon to be after it stops in the Pb for 𝜇! and for 𝜇!? [10 points]

(b) What type of particles are emitted after the muon stops for 𝜇! and for 𝜇!? [10 points]

(c) What are the energy ranges of the particles emitted in item (b)? Give the answer in terms of eV or MeV for 𝜇! and for 𝜇!. [10 points]

(d) What is the lifetime of the muon in its own frame compared to it decaying in vacuum? Do not calculate a number but state if it is the same, a shorter, or a longer time and why for 𝜇! and for 𝜇!. [10 points]

masses: muon 105 MeV electron 0.5 MeV pion 135 and 140 MeV

proton 938 MeV neutron 940 MeV photon and neutrino 0

Page 38: Ph. D. Qualifying Exam Jan. 2017 Modern and Statistical Physics

Problem 5: (40 points) Classical Thermodynamics of a Two-state System: A (very small) discrete system has only two states 1 and 2 with energies 𝐸! = −𝜀! and 𝐸! = 𝜀!, respectively. This could, for instance, be a spin 1 2 particle in an external magnetic field. Since this system contains only one particle, the different thermodynamic ensembles (of the systems described below) do not provide equivalent descriptions of the physics. We want to explore this difference for this simplest possible system.

(a) If the system is isolated from the environment which are the possible values for the internal energy of the system? [5 points]

(b) In the following we assume that the system is not isolated any more but instead interacting with a heat bath of temperature 𝑇. Using the canonical distribution of classical thermodynamics, what are the probabilities 𝑝! to find the system in each of the two states in this case? [10 points]

(c) Find the internal energy as a function of the temperature of the heat bath (express it in terms of a familiar hyperbolic function). [15 points]

(d) Using the limiting values of the expression in (c), what are the possible values for the internal energy of the system when coupled to the heat bath? [10 points]

Page 39: Ph. D. Qualifying Exam Jan. 2017 Modern and Statistical Physics

Ph. D. Qualifying Exam Feb. 2012

Modern and Statistical Physics Do 3 out of 5 problems Problem 1: (40 points) 1. Phase diagram, the critical point: The equation of state of van der

Waals gas is P av2

v b RT , where a and b are characteristic

constants for a given gas, and P, v, T and R are pressure, specific volume, temperature and gas constant, respectively. When we regard the above van der Waals equation as P = P(v, T)

(a) what are the three conditions at the critical point in the critical

isotherm (T = TC) on a P-v diagram? [15 points]

(b) Express the critical specific volume, vC, the critical temperature, TC, and the critical pressure, PC, at the critical point, in terms of the constants a and b. [25 points]

Problem 2: (40 points) A metal ring is dipped into a soapy solution (index of refraction ) and held in a vertical plane so that a wedge-shaped film formed under the influence of gravity. At near-normal illumination with blue-green light (wavelength ) from an argon laser, one can see fringes per cm. Determine the wedge angle of the soap film. (Note: assume that the wedge angle is very small).

Page 40: Ph. D. Qualifying Exam Jan. 2017 Modern and Statistical Physics

Problem 3: (40 points) 2. A pi-mu atom consists of a pion and a muon bound in a Hydrogen-like

atom. (a) What are the energy levels for such an atom compared to those for

Hydrogen expressed in terms of the electron, pion, and muon masses? [20 points]

(b) The pi-mu atoms are produced in decays pi-mu atom neutrino. If the has , what are the minimum and maximum energies

of the pi-mu atom in the moving frame of the ? (express these energies in terms of the particle masses) [20 points]

Problem 4: (40 points) Briefly explain or describe 4 of the following 6 phenomena (in no more than 200 words for each phenomena): [10 points each]

(a) Electromagnetic structures of the neutron and proton

(b) The laser

(c) C, P, T, and CP symmetry (and any possible well known violations)

(d) The transistor

(e)

(f) Superconductivity

Page 41: Ph. D. Qualifying Exam Jan. 2017 Modern and Statistical Physics

Problem 5: (40 points) A system consisting of N (a very large number) identical weakly interacting particles is in equilibrium with a heat bath. The total number of individual states available to each particle is 2N. Of these, N states are degenerate with energy 0, and N states are degenerate with energy . It is found by observation that the total energy of the system is . Find the temperature of the heat bath under three different assumptions:

(a) That the particles are bosons. [10 points]

(b) That the particles are fermions. [10 points]

(c) That the particles obey the (unphysical) Boltzmann distribution. [10 points]

(d) You should find from Parts (a), (b), and (c) that

Explain why this is so. [10 points]

0

N states

N states

Page 42: Ph. D. Qualifying Exam Jan. 2017 Modern and Statistical Physics

Modern and Statistical Physics Fall 20119/24/2011

Do 3 out of 5 problems

1. Classical Thermodynamics Ideal Gas [40 points]

Assume that the earth’s atmosphere is an isothermal ideal gas in a gravitational field.Consider a thin layer of the ideal gas at height z and of thickness dz. There is a difference ofpressure across this thin layer, dP, which must just balance the gravitational force on themass. If the pressure at z = 0 is P0 , determine the pressure as a function of height z.

2. The Kinetic theory of Gas [40 points] The ditribution of particle speeds of a certainhypothetical gas is given by

N v( )dv = Ave−v v0dv ,

wherer A and v0 are constants.

a. Determine A so that f v( ) ≡ N v( ) N is a true probability density function; i.e.,

f v( )dv0

∫ = 1 . [10 points]

b. Find the mean speed, v , and the root mean square speed, vrms , in terms of v0. [10 points]

c. Find the most probable speed vm. [10 points]

d. What is the standard deviation of the speeds from the mean, σ, in this case? [10 points]

3. Solid state physics, thermodynamics, statistical mechanics [40 points] There is asemiconductor with two bands: The lower band is described by

Ek( ) , the upper band by

E0 − E

k( ) . The two bands are separated by an energy gap of 2Δ. (Hint: Use electron-hole

symmetry. This is also evident from the way the upper band is defined.)a. Calculate the chemical potential as a function of the temperature. [20 points]

Fig.1

Page 43: Ph. D. Qualifying Exam Jan. 2017 Modern and Statistical Physics

b. Examine whether the system can be treated using classical statics at low temperatures.[20 points]

4. High energy physics [40 points]a. A particle of mass m is produced with energy E and decays after traveling a distance l.

How long did the particle live in its own rest frame? [8 points]

b. Use the result of part (a) to determine the spatial separation between the production anddecay vertices (a.k.a. “decay length”) of a B0 meson carrying an energy of 65 GeV, if itlived 1.5 ps in its own rest frame. The mass of the B0 meson is 5.3 GeV. [8 points]

c. Determine the maximum energy that can be carried off by any one of the decay particles,when a particle of mass m0 at rest decays into three particles with masses m1, m2, and m3.[24 points]

5. Diffraction grating (Multiple slit diffraction) [40 points] Consider diffraction of an array ofmultiple slits (N equal slits of width d and common spacing of a, numbered from 0 to N - 1) ,or diffraction grating, illuminated by monochromatic plane wave normal to the array (Fig 2).We assume that the length of the slit is long enough, and the obervation point P is far enoughfrom the slit.

a. Obtain the expression for the intensity of the multiple slit diffraction pattern at a point P.[20 points]

b. Now using the above diffraction grating, we perform an experiment to determine thewavelength of light. When the wavelength of the light λ changes to λ + dλ, thediffraction pattern shifts. If this shifts is smaller than the width of a bright band of thediffraction pattern, the wavelength λ and λ + dλ cannot be distinguished one another.Determine the accuracy of the measurement of the wavelength by this experimentalmethod. [20 points]

Fig.2

Page 44: Ph. D. Qualifying Exam Jan. 2017 Modern and Statistical Physics

Modern and Statistical Physics Spring 20112/26/2011

Do 3 out of 5 problems

1. High energy physics [40 points]Consider the reaction p + gamma --> neutron + pi+.

a. Assume that the gammas are the cosmic blackbody radiation with average kT = 3×10−4

eV. What is the threshold energy that the proton must have for the reaction to occur? [20points]

mp ~ mn ~ 1000 MeV, mπ = 140 MeV, mµ = 105 MeV, mν = 0 eV.

b. The pion then decays to µ + ν. In the pion rest frame, what is the neutrino energy? Whatis the neutrino energy in the "lab" frame? This is the source of the highest energyneutrinos in the universe. Give answers in eV units. [20 points]

2. Relativity C-O white dwarf [40 points]

a. A C-O white dwarf has radius R and mass M. Assuming the electrons are degenerate,what is the average energy of the electrons in terms of M, R and fundamental constantsassuming the electrons are non-relativistic? [15 points]

b. Repeat assuming the electrons are relativistic. [15 points]

c. Now assume that the radius shrinks by a factor of 2 so R' = R/2 while the mass remainsthe same. What is the relative change in the gravitational and electron energies(assuming both relativistic and non-relativistic) between R and R'? [5 points]

d. Is the star more stable (less likely to collapse) if the electrons are relativistic or non-relativistic? Why? [5 points]

3. 2D harmonic oscillator [40 points]

Give a 2D harmonic oscillator with Hamiltonian, H =p

x

2

2m+p

y

2

2m+12mω 2 x2 + y2( ) + kmxy .

a. For k=0, what are the energies of the ground state and first and second excited states?What are the degeneracies of each state? [20 points]

b. For k>0, using first order perturbation theory, what are the energy shifts of the groundstate and the first excited states? [20 points]

Page 45: Ph. D. Qualifying Exam Jan. 2017 Modern and Statistical Physics

4 Classical mechanics Van der Waals Gas [40 points]The Van der Waals equation of state for one mole of a non-ideal gas reads

p +aV 2

⎛⎝⎜

⎞⎠⎟V − b( ) = RT .

[Note: part (d) of this problem can be done independently of part (a) to (c).]a. Sketch four isotherms of the Van derWaals gas in the p-V plane (V along the horizontal

axis, p along the vertical axis). Identify the critical point. [10 points]b. Evaluate the dimensionless ratio pV/RT at the critical point. [10 points]

c. In a portion of the p-V plane below the critical point, the liquid and gas phases cancoexist. In this region the isotherms given by the Van der Waals equation are unphysicaland must be modified. The physically correct isotherms in this region are lines ofconstant pressure, p0(T). Maxwell proposed that p0(T) should be chosen so that the areaunder the modified isotherm should equal the area under the original Van der Waalsisotherm. Draw a modified isotherm and explain the idea behind Maxwell's construction.[10 points]

d. Show that the heat capacity at constant volume of a Van der Waals gas is a function oftemperature alone (i.e., independent of V). [10 points]

5. Statistical mechanics [40 points]There is a system of two identical particles, which may occupy any of the three energy levelsε0 = 0, ε1 = ε, ε2 = 3ε.

The system is in thermal equilibrium at Temperature T, which means that the system is acanonical ensemble. For each of the following cases, determine the partition function and theenergy and carefully enumerate the configurations.

a. The particles are Fermions. [10 points]b. The particles are Bosons. [10 points]

c. The particles obey Boltzmann statistics and now they are distinguishable. [10 points]d. Discuss the conditions under which Fermions or Bosons may be treated as Boltzmann

particles. [10 points]

Page 46: Ph. D. Qualifying Exam Jan. 2017 Modern and Statistical Physics

Modern and Statistical Physics. September/October 2010 Do 3 of 5 problems

1. A top quark decays to a bottom quark and a W boson. The W boson then decays to a positron and aneutrino. Give your answers below in terms of the masses mt , mb , mW of the top quark, the bottom quark,and the W boson respectively. You may treat the positron and the neutrino as massless. You areencouraged to set c=1.

(a) (14 points) What is the momentum of the bottom quark, in the rest frame of the top quark?

(b) (14 points) What is the maximum possible momentum of the positron, in the rest frame of the topquark?

(c) (12 points) Suppose the top quark has speed 0.5c in the lab frame, before it decays. What is themaximum possible momentum of the positron in the lab frame?

2. A simple model of a rubber band is a one­dimensional (horizontal) chain consisting of N (N >> 1)linked segments, as shown schematically in the diagram. Each segment has two possible states: horizontal(H) with length a, or vertical (V), with length zero; with the figure indicating a state of HHHH­VHVH­VHHH­HVHV­H. The segments are linked such that they cannot come apart. The chain is in thermalcontact with a reservoir at temperature T.

(a) (10 points) If there is no energy difference between the two states, what is the average length of thechain?

(b) (25 points) Fix the chain at one end and hang weight from the other end, supplying a force F as shown.Determine the average length of the chain at any temperature T and show that it is equivalent to

⟨L T ⟩=aNeFa /2 kT

e−Fa /2 kTeFa /2 kT

.

Page 47: Ph. D. Qualifying Exam Jan. 2017 Modern and Statistical Physics

(c) (5 points) In which temperature limit is the extension proportional to F (Hooke's law)? Calculate theconstant of proportionality (that is the “k” from Hooke’s law).

3. Artificial materials (or metamaterials) can be engineered to provide a negative index of refraction n<0.In this problem we explore the amazing properties of two arrangements of metamaterials described in theFigure below.

1. (6 points) Consider an incoming optical ray at the interface from vacuum to a metamaterial. Writedown the expression for the refracted angle and draw the incoming and refracted ray for n=­1.Indicate the difference(s) with the result obtained for conventional materials.

2. (17 points) We now consider a slab of metamaterial with thickness t as drawn in the Figure (a)with n=­1.

a. (6 points) Let a point­like source object be at a distance d=t upstream of the slab, showthat there is an image and give the location of this image.

b. (6 points) Same question if the object is at a distance d=1/2t. c. (5 points) Comment on the properties of a simple slab with negative index of refraction

compared to a standard (n>0) slab. 3. (17 points) Consider the configuration shown in Figure (b).

a. (6 points) Via geometrical construction, show that the source is imaged on the lower­right quadrant.

b. (6 points) Show that rays emitted by the source are also imaged on the source itself. c. (5 points) Discuss possible applications of such imaging/recirculating configurations.

4. In the Big Bang theory of the universe, the radiation energy initially confined in a small regionadiabatically expands in a spherically symmetric manner. The radiation cools down as it expands.

Page 48: Ph. D. Qualifying Exam Jan. 2017 Modern and Statistical Physics

a. Derive a relation between the temperature T and the radius R of the spherical volume ofradiation, based purely on thermodynamic considerations. Here the radiation pressure isexpressed as p = U 3V , and the black body radiation energy density isu = U V = aT 4 . [20 points]

b. Find the total entropy of a phonon gas as a function of its temperature T, volume V andthe constants k,h,c . [20 points]

5. A flashlight is collimated so that its beam goes off in a 90 degree cone in its own rest frame.

a) (20 points) Calculate the opening angle of the flashlight cone when the flashlightis moving forward at velocity v.

b) (20 points) Assuming that 2 21/ 1 / 1v cγ = − >> show that the opening angle ofthe cone is about 1/ γ

Page 49: Ph. D. Qualifying Exam Jan. 2017 Modern and Statistical Physics

Modem and Statistical Physics. February 2010Do 3 of 5 problems

1. Consider the reaction nubar * p ) e+ + n.a. What is the threshold energy for the antineutrino if the reaction occurs off Hydrogen? Give answer

inMeV.b. If the reaction occurs off Carbon 12 (with proton density of 0.1/F^3), the reaction will be suppressed

at low antineutrino energies. Explain why. (hint, note the two lowest spectroscopic states in nuclei.)c. Assuming a Fermi gas model, what is the Fermi energy for protons nC 12? What is the average

proton kinetic energy? Give in MeV.d. Up to what antineutrino energy will suppression in C 12 still be a factor?

mass neutron : g3g.6MeYlc2 hbar*c : 197 MeV*F hc : 1240 MeV*Fmass proton: 938.3 MeV/c2mass electron: 0.5 MeV/c2

2. Forthis problem, do not calculate any exponentials or roots but put in all the other numbers for the finalanswer

a. For a H atom, what is the ratio of the probability to be in the n:2Ievel compared to the n:l level atT : 3000 degrees K? E(lS) -- -13.6 eV and kT: .025 eV at T : 300 degrees K

b. For a metal with a Fermi energy equal to 4 eV and at T : 300 degrees K determine the followingassuming a Fermi gas model:

i. Wfrat is the density for conduction electrons? hc: 1240 eV*nm mass electron : 0.5 MeV/c2ii. What is the ratio of the density of states between 4.4 eY and 4.0 eV?iii. What is the ratio of the probability to have the energy be 4.4 eY compared to 4.0 eV?

c. For a photon in a Bosonic gas at T : 300 degrees K:i. What is the ratio of the density of states between 4.4 eY and 4.0 eV?ii. What is the ratio of the probability to have the energy be 4.4 eV compared to 4.0 eV?

3. A rocketship is traveling towards the Earth at 0.6c. It hres a projectile of mass M with a velocity of 0.8cin the rocketship's frame. What is the energy of the projectile in the Earth's frame? If the rocket ship istraveling perpendicular to a line from the ship to the Earth and again fires a projectile at 0.8c in the ship'sframe and uf 90 d.gr""s from the direction of the ship's motion (in the rocketship's frame), what is theenergy of the projectile in the Earth's frame? What is the tangent of the angle of the projectile relativeto the ship's motion (don't calculate but give as the ratio of two components)?

4. Optical mirage:We explore the propagation of an optical ray in the (Oxz)plane; see Figure. We assume the refractive index n obeys

the relation z = an2 * 6 where a andb are constants.a. (10 points) Show that at a given height z we have

n(z)cosd:floaosd,o=A where A is a

constant.b. (10 points) Deduce the differential equation for the

ray trajectory (#)' =(1)'- l and show that

the resulting ray trajectory is a pardbola.c. (20 points) We now wish that the model discussed

in (f and (2) canbe used to explain the formation of optical mirages. We consider the (Oxz) planeto be filled with an ideal gas. We assume the gas to be in mechanical equilibrium (so that thechange of pressure dP depends on the height variation dz as dP=-pgdz where g is the

Page 50: Ph. D. Qualifying Exam Jan. 2017 Modern and Statistical Physics

gravitational constant and pthe gas density). We consider a temperature gradient along z of the

form T(z) : T0(l - kz)where k is a constant and firther assume the refractive index and the gas

density satisff the relation trn'z -tIt p = const .

(15 points) Show that the refractive index is related to z via z : an2 -l b and explicitlydetermine the constants a and b.(5 points) Qualitatively describe the formation of a "warm" mirage due to a hot floor.

5. Partition function of an Einsteinsolid is

7_ e "'t - t-'-en'r 'where ft is the Einstein temperahtre. Treat the crystalline lattice as an assemblyof 3N distinguishable oscillators.

a. Calculate the Helmholtz function F. [10 points]b. Calculate the Enhopy,S. [10 points]c. Show that the entropy approaches zero as the temperature goes to absolute zero. [10

pointsld. Find the entropy at high temperatures. [10 points]

Page 51: Ph. D. Qualifying Exam Jan. 2017 Modern and Statistical Physics

Modern and Statistical Physics. September 2009Do 3 of 5 problems

I . The face-centered cubic (fcc) is the most dense and the simple cubic (sc) is the least dense of the threecubic Bravais lattices. Suppose identical solid sphere are distributed through space in such a way that theircenters lie on the points ofeach ofthese three structures, and spheres on neighboring pointsjust touch,without overlapping. (Such an arrangement of sphere is called a close-packing arrangement)a). Assuming that the spheres have unit density, show that the density ofa set ofclose-packed spheres on

eachofthetbreestructures(the"packingfraction") ," tlinl6x0.l4 forfcc, ",linl8r0.68 forbody-centered cubic (bcc), and n l6 p 0.52 for sc.b). Show that for a fcc Bravais lattice the free electron Fermi sphere for valence 3 extends beyond point Wof the first Brillouin zone (see figure), so that the first Brillouin zone is completely frlled fHint: provekF lf W = (1296 ll25 o')t' u = 1.0081

2. Considerthe reactiony+ e ) r| +n- +ewiththe electroninitiallyfree andatrest (withthis framedesignated as the lab frame).

a) What is the minimum photon energy for this reaction to proceed?b) Assuming that the photon energy is the minimum determined in a), what is the velocity of either of

the pions after the reaction?c)Assumethepionthendecaystoamuonandneutrinon)V+v.Inthelabframe,whatisthe

maximum energy of the muon produced in the pion decay assuming the photon energy determined in a).Give your answers in terms of the pion, muon, and electron masses while setting the photon and neutrino

masses to 0.

3. A 55 year old man can focus objects clearly from 100 cm to 300 cm. Representing the eye as a simplelens 2 cm from the retina,a) what is the focal length of the lens at the far point (focused at 300 cm)?b) what is the focal length of the lens at the ner point (focused at 100 cm)?c) what strength lens (focal length) must he wear in the lower part of his bifocal eyeglasses to focus at 25cm?

Page 52: Ph. D. Qualifying Exam Jan. 2017 Modern and Statistical Physics

4. A smooth vertical tube having two different sections is open from both ends and equipped with twopistons of different areas. Each piston slides within its respective tube section. One mole of ideal gas isinclosed between the pistons. The pistons are connected by a non-stretchable rod. The outside air pressureis 1 atm. The total mass of the pistons is M. The cross sectional area of the larger upper piston A1, and the

lower piston A2 are related by 4 : A" + AA. Find the rise in the temperature of the gas between the

pistons required to lift the piston assembly by a distance L?

5. Assume that the neutron density in a neutron star is 0. l/frn3 lthat is 0. I neutron percubic Fermi). Assuming T:0, ignoringany gravitational forces, and using a Fermi gas model with uniformdensity determinea) the average energy ofthe neutronsb) the average energy ofelectrons ifthe electron density is 1% ofthe neutron densityc) show two reactions, one which can convert a neutron to a proton and one which can convert a proton

to a neutrond) determine the neutron to proton ratio. Hint, consider the Fermi energies of the neutrons, protons and

electrons at equilibriumGive answers in i) and b) in MeV using hc : 1240 MeVfrn, the mass of the neutron: 1000 MeV/c2, and

the mass of the electron : 0.5 MeV/c2. You will need to decide if the particles are relativistic or non-relativistic. The answer for d) can be given in terms of the particle masses.

Page 53: Ph. D. Qualifying Exam Jan. 2017 Modern and Statistical Physics

February 2009. Stat/Modern/Thermo PhD. Qualifier. Pick 3 questions out of 5.

1. a. (20) Show using conservation of energy and momentum that it is not possible for a free

electron moving through vacuum to emit a photon. b. (20) Now consider the related problem of an electron moving through superfluid

helium. Show that in this case the electron can emit a phonon as long as it moves with a

velocity exceeding a critical velocity cv and find cv . The excitation spectrum of the

phonons in superfluid helium is given by E up= where u is the velocity of sound and p

is the momentum of the phonon. You may do this part in the non‐relativistic limit. 2.

a. (10) A C‐O white dwarf has a radius = R, mass = M and is assumed to have constant density. Assuming the electrons are degenerate and non‐relativistic find the average energy of the electrons in terms of M, R and fundamental constants.

b. (10) Repeat part a) assuming the electrons are relativistic.

c. (10) Assume the radius shrinks by a factor of 2 so / 2R R′ = while the mass remains the same. Find the relative change in the gravitational and electron energies (assuming both relativistic and non‐relativistic) between R and R’.

d. (10) Discuss whether the star is more stable (less likely to collapse) if the electrons are relativistic or if they are non‐relativistic.

3. When a large number of atoms come together to produce a solid each atomic level broadens into a band. Draw a picture of simplified band structure, define the valence and conduction bands, and describe the temperature‐dependent behavior of conductivity for:

a. (10) Metals (define the term Fermi energy) b. (10) Insulators (define the term band gap) c. (10) Intrinsic semiconductors d. (10) The n‐type and p‐type semiconductors

4. One mole of a monatomic ideal gas initially at temperature 0T expands from volume 0V

to 02V . Calculate the work of expansion and the heat absorbed by the gas for the case

of expansion at: a. (20) Constant temperature

b. (20) Constant pressure

5. a. (20) Consider a large number of N localized particles in an external magnetic

field H (directed along the z direction). Each particle has a spin s = 1/2. Find the

number of states accessible to the system as a function of sM , the z component

of the total spin of the system. Determine the value of sM for which the number

of states is maximum.

Page 54: Ph. D. Qualifying Exam Jan. 2017 Modern and Statistical Physics

b. (20) Define the absolute zero of the thermodynamic temperature. Explain the meaning of negative absolute temperature, and give a concrete example to show how the negative absolute temperature can be reached.

Page 55: Ph. D. Qualifying Exam Jan. 2017 Modern and Statistical Physics

September 2008. Stat/Modern/Thermo PhD. Qualifier. Pick 3 questions out of 5.

1. The electronic structure of the atom is described by four quantum numbers. a. Define these quantum numbers and describe the values they can acquire. b. Write down the electronic configurations for elements with atomic numbers: 8

(Oxygen), 19 (Potassium), 29 (Copper). c. Define the term atomic valence and find the valence for the elements O, K, and

Cu. d. List four main types of bonding found in materials and give examples of the

elements and/or compounds for each type of bonding. 2. Consider the reaction p e nν ++ → + .

a. Find the threshold energy for the ν if the reaction occurs off Hydrogen. b. Explain why the reaction will be suppressed due to Pauli Exclusion if it occurs

off 12 C (with proton density of 0.1/F3). c. Assuming a Fermi gas model, find the approximate suppression as a function of

neutrino energy. ( 2940MeV/cnm = , 197c = MEV F, 1240hc = MeV F)

3. A parallel beam of electrons is directed through a narrow slit of width a and a screen is placed at a distance d from the slit. The first diffraction minimum is observed at a distance y from the central maximum.

a. Find the velocity of the electrons in terms of a , d , y , and em assuming the electrons are non-relativistic.

b. Find the velocity assuming the electrons are relativistic. c. Estimate the spread of the electron beam through the slit using the Heisenberg

uncertainty principle and show that this is consistent with the spread estimated based on the width of the central maximum.

Page 56: Ph. D. Qualifying Exam Jan. 2017 Modern and Statistical Physics

4. We have an ensemble of N spins that have a Zeeman splitting described by the

Hamiltonian

.=1=

σµ BH B

N

i∑− (1)

Here B is the magnetic field, Bµ is the Bohr magneton, and 1= ±σ is the direction of the spin.

a. Show that the partition function is given by

.)cosh(2= N

B

B

TkBZ µ (2)

b. Show that the energy in terms of the partition function is given by

T

ZTkE B ∂∂ ln= 2 (3)

and the magnetization by

B

ZTkM B ∂∂ ln= (4)

c. Determine E and M for the partition function in Eqn. (2) and sketch them as a function of temperature.

5. Chemical potential of an ideal gas.

a. Starting with TdS dU PdV= + show that the chemical potential of an ideal gas can be written in terms of the temperature T and the volume V as :

0ln( ) ln( ) constantP Vc T c T T RT V S Tµ = − − − + .

Here, cP is the specific heat constant pressure, cv is the specific heat at constant volume and 0S is a constant.

b. Starting from TdS dH VdP= − find a similar expression for the chemical potential but now as a function of T and P.

c. Show that the chemical potential at the fixed temperature T varies with pressure:

µ = µ0 + RT ln PP0

.

Here, µ0 is the value of µ at the reference point (P0, T)

Page 57: Ph. D. Qualifying Exam Jan. 2017 Modern and Statistical Physics

Stat/Modern/Thermo PhD. candidacy examination. March 2008

Pick 3 questions out of 5.

1) Consider a string of length L , mass density µ and string tension τ. The string ismaintained at finite temperature T .

a. Find the average amplitude of a mode of the string of wavelength

λ = 2L n in thelimit of

kBT >> hf , with f the frequency of the mode.b. Explain why it is necessary to state that

kBT >> hf .

2) Assume that the crystal lattice structure of solid comprising N atoms can be treated as anassembly of 3N distinguishable one-dimensional oscillators (Einstein solid).

a. What is the partition function Z? Use the Einstein temperature θE (≡ hν/k, where νis natural frequency).

b. Calculate the Helmholtz function F.c. Calculate the entropy S.d. Show that the entropy approaches zero as the temperature goes to absolute zero.

3) The variation of the internal energy U as a function of entropy S is predicted by classicalequilibrium thermodynamics to have a functional form of

U −U0 =α S − S0( )n where α isa constant for a fixed value of volume.

a. Show that the temperature as a function of entropy in the case of constant volumeis given by

T =αn S − S0( )n−1.

b. Show that

Cv =S − S0n −1

=1n −1

Tαn

1 n−1

.

At some point it may be useful to derive the relationship

∂T∂S

V

=∂ 2U∂S2

V

.

4) Silicon has Z=14.

a. Relative to a hydrogen atom, what are the energy and radius of an electron in the1s shell?

b. In the 3p shell?c. Assume that there are 2 electrons in the 3p shell. What are the allowed

spectroscopic states for this 2-electron system? (Give S, L, J).d. What is the energy ordering and why?

5) A charged kaon (at rest) decays by

K + →π + + π 0 and then

π 0 →γ + γ .

a. In terms of the masses of the particles what is the 0π ’s energy?b. What is the maximum photon energy?

Page 58: Ph. D. Qualifying Exam Jan. 2017 Modern and Statistical Physics

Modern/Optics/Thermodynamics September 2007

Pick 4 out of 6 problems.

1 In inertial frame O a rod of length l is oriented along the x-axis and moving withvelocity u in the positive y direction. This rod is then viewed from an inertial referenceframe O’ moving with velocity v in the positive x direction.

a) What is the length of the rod in O’? [10 points]b) What angle does the rod make with respect to the x’ axis? [30 points]

2 A cubic box (with sides of length L) holds diatomic H2 gas at temperature T. Each H2

molecule consists of two hydrogen atoms with mass of m each separated by distance d.Assume that the gas behaves like an ideal gas. Ignore the vibrational degree of freedom.

a. What is the average velocity of the molecules? [10 points]b. What is the average velocity of rotation of the molecules around an axis which is

the perpendicular bisector of the line joining the two atoms (assuming each atomas a point mass)? [10 points]

c. Derive the expressions expected for the molar heat capacities Cp and Cv for such agas. [10 x 2 points]

3.a. What is the threshold kinetic energy for the proton for p+ n p+ p+ π− assuming

the neutron is at rest? [20 points]b. If now the neutron is in a Carbon nuclei of size 60 F3 (with 6 protons and 6

neutrons), what is the Fermi energy of the neutron, and what is the thresholdkinetic energy in this case? [20 points]

mn = mp = 1000 MeV, mπ = 140 MeV,

hc = 200MeVF

4. A smooth vertical tube having two different sections is open from both ends andequipped with two pistons of different areas. Each piston slides within its respective tubesection. One mole of ideal gas is enclosed between the pistons. The pistons are connectedby a non-stretchable rod. The outside air pressure is 1 atm.

The total mass of the pistons is M. The cross sectional area of the larger upper piston A1,and the lower piston A2 are related by A1= A2+ΔA.

How much (in Kelvin) must the inner gas (between the pistons) be heated in order to liftthe piston assembly by L=5cm?

Page 59: Ph. D. Qualifying Exam Jan. 2017 Modern and Statistical Physics

β

biprism

5. Eight non-interacting neutrons are confined to a 3D square well of size D= 5 F (10-15

m) such that V = -50 MeV for0 < x < D,0 < y < D,0 < z < Dand V = 0 everywhere else.

a. How many energy levels are there in this well? [10 points]b. What is the degeneracy of each energy level? [10 points]c. What is the approximate Fermi energy for this system? [10 points]d. What is the relative probability to be in the lowest energy state to the fourth lowest

energy state at kT= 10 MeV? Just write down the ratio (don't calculate the value).[10 points]

mn = 1000 MeV,

hc = 200MeVF

6. Interference by a biprism. A plane wave (wavelengthλ) enters perpendicular to the biprism (the prism angleα, refractive index n) as shown in the figure. The wavetransmitted through both sides of the biprism is bent(refracted) and overlap at the viewing screen S (parallelto the biprism) where an interference pattern can beobserved.

a. Find the refracted angle, β. [20 points]b. Find the interval of adjacent interference lines.

[20 points]

one moleideal gas

p0

p0

M=5kgp0=1atmA1-A2=ΔA=10cm2

CONSTANTS:

g=9.8 m/s2

R=8.3J/(oK⋅mole)kboltz=1.38x10−23J/oK

Page 60: Ph. D. Qualifying Exam Jan. 2017 Modern and Statistical Physics

Statistical and Modern. Spring 2007. Pick 4 out of 6.

1) a) Starting with the first law of thermodynamics and the definition of pc and vc , showthat

p vT P

U Vc c p

V T

∂ ∂ − = + ∂ ∂ .

Here pc and vc are the specific heat capacities per mole at constant pressure and volumerespectively, and U and V are the energy and volume of one mole.

b) Use the above result plus the expression

T V

U pp T

V T∂ ∂ + = ∂ ∂

to find p vc c− for a van der Waals gas with equation of state

( )2

ap V b RT

V + − =

.

Here a and b are constants.

c) Use this result to show that as V → ∞ at constant p , you obtain the ideal gas result for

p vc c− .

2) The rotational motion of a diatomic molecule is specified by two angular variablesθand φ and the corresponding canonical conjugate momenta, ,p pθ φ . Assuming the form ofthe kinetic energy of the rotational motion to be

( )2 2

21 12 2 sinrot p p

I Iθ φεθ

= +

a) Derive the classical formula for the rotational partition function , ( ),r T

22

( )IkT

r T =h

b) Calculate the Helmholtz free energy rotF .c) Calculate the corresponding entropy and specific heat.

The following may be helpful

e dxa

dxax

ax a

ax−

−∞

=

= −

2

2

π

sin ( )cot( ) /

3) Assume that the neutron density in a neutron star is 30.1/fm (that is 0.1 neutron percubic Fermi). Assuming T=0 and ignoring any gravitational forces calculate the ratio ofneutrons to protons to electrons.

Page 61: Ph. D. Qualifying Exam Jan. 2017 Modern and Statistical Physics

Hint: determine their Fermi energy. The electron, neutron and protons masses are.511 2MeV/c , 939.6 2MeV/c and 938.3 2MeV/c . The constant 1240MeV fmhc = . Youshould be able to work out "by hand" an approximate value.

4) A π µ− atom consists of a pion and a muon bound in a Hydrogen-like atom.a) What are the energy levels for such an atom compared to those for Hydrogen?

b) π µ− atoms are produced in KL decays ( LK π µ ν→ − + ). If the KL has 0.8β = whatare the minimum and maximum energies of the π µ− atom expressed in terms of the K ,π and µ masses with 0mν = ?

c) Approximately what fraction of KL decays will produce a π µ− atom (hint: use theHeisenberg uncertainty principle)?

5) a) You are familiar with the quarter-wave thin film coating that acts as a “reflection–reducer”.For the moment, let us look at a simpler thin film—the air gap between two pieces of glass suchas you would find in a Newton’s rings experiment. Why do we get constructive interference inthe reflected when the thickness is one-fourth of the wavelength of light or some odd multiple ofa quarter wavelength? Why isn’t it constructive at one-half wavelength of the light? Forassistance, I present two of the Fresnel equations (in two forms) for reflected light.

cos cos tan( )cos cos tan( )

cos cos sin( )cos cos sin( )

t i i t i t

t i i t i t

i i t t i t

i i t t i t

n nr

n n

n nr

n n

θ θ θ θθ θ θ θ

θ θ θ θθ θ θ θ⊥

− −= =

+ +

− −= = −

+ +

P

Where the parallel and perpendicular symbols refer to the plane of incidence, and i,t refer toincident and transmitted media, θ ’s are angles of incidence and transmission, and n’s are indicesof refraction.

b) In light of the previous, to get destructive reflection in a thin film-i.e.-a quarter-wave film, suchas the one illustrated below, what condition must prevail among the indices of refraction for thethree media (n0 may be taken as = 1.0 for air.)

c) The destructive interference described in part b) will generally not be complete. Find the valueof 1n as a function of 2n which gives completely destructive interference at normal incidence.

n1

n2

n0

Page 62: Ph. D. Qualifying Exam Jan. 2017 Modern and Statistical Physics

6) In a big-bang theory of the universe, the radiation energy initially confined in a smallregion adiabatically expands in a spherically symmetric manner. Here the radiation(photon) pressure is expressed as p = U 3V , and the black body radiation energy densityis u = U V = aT 4 The radiation cools down as it expands.

a) Derive a relation between the temperature T and the radius R of the spherical volumeof radiation, based purely on thermodynamic considerations.

b) For the above problem, show the total entropy of a photon gas is expressed as34

3S aT V= .

Page 63: Ph. D. Qualifying Exam Jan. 2017 Modern and Statistical Physics

Modern Physics, Optics, Statistical Mechanics and Thermodynamics September, 2006 Pick 4 out of 6 problems

1) A flashlight, in its own rest frame, is directed at an angle θ of 45 degrees to the x-axis in the xy plane. What angle will the light beam appear to make to an observer moving towards the flashlight along the x-axis at velocity β ?

a. Now consider a relativistic electron traveling in a circular orbit. Explain why the radiation from the electron will be confined to a narrow cone, and calculate the opening angle of the cone.

b. 2) A simple model of a rubber band is a one-dimensional (horizontal) chain consisting of N ( 1N ) linked segments, as shown schematically in the diagram.

Each segment has two possible states: horizontal with length a, or vertical, contributing nothing to the length. The segments are linked such that they cannot come apart. The chain is in thermal contact with a reservoir at temperature T.

a. If there is no energy difference between the two states, what is the average length of the chain? b. Fix chain at one end and hang weight from the other end, supplying a force F as shown. Determine the average length of the chain at any temperature T . Find the length in the limits 0T → andT →∞ .

c. In which temperature limit is the extension proportional to F (Hooke’law)? Calculate the constant of proportionality.

3) The complete formula for multislit diffraction pattern in Fraunhofer diffraction is given by the expression

2 2

0sin sin( )

sinNI I β αθ

β α =

where 0I is the flux density in the 0θ = direction for one slit,

Page 64: Ph. D. Qualifying Exam Jan. 2017 Modern and Statistical Physics

sin ,2kbβ θ= and b is the slit width,

sin2kaα θ= , and a is the slit separation, and

2k πλ

= is the propagation number.

a. Show that 2

0(0)I N I= , where N is the number of slits. b. For 3N ≥ there are secondary maxima between the principal maxima. Deduce the rule for the number of minima between principal maxima, and, then the number of secondary maxima. Actually deduce this, don’t just recite it if you happen to remember it! c. The result in a indicates that the flux density at 0θ = varies as 2.N Explain how that can be true. After all, e.g., if you have three slits, then there is three times a much light. How can the flux density be nine times as great?

4) Consider a system of two types of charge carriers in the Drude model. The two carriers have different densities (n1 and n2) and opposite charge (e and –e), and their masses and relaxation times m1, m2 and τ1, τ2, respectively. Determine the Hall coefficient RH of this system.

5) Assume that the cross section ratio for the two body reactions given below depends only on phase space of the final state particlesLarine o . Calculate the ratio.

e e

p pµ µ

µ µ

ν ν

ν ν

+ → +

+ → +

6). Consider an infinitely long cylinder that has been cut in half, with the two halves thermally insulated from each other (see figure). The top half is held at temperature T=30ºC, and the bottom half at T=10ºC. Find the temperature T(x,y) inside the cylinder. Hint: Use the conformal mapping

1ln1

zwz

z x iy

+ = − = +

Page 65: Ph. D. Qualifying Exam Jan. 2017 Modern and Statistical Physics

Modern Physics, Optics, Statistical Mechanics and ThermodynamicsFebruary, 2006

Pick 4 out of 6 problems

1. (Bohn) Consider an electron beam of circular cross section moving down the z-axisand passing through a system of focusing magnets. Suppose the beam densityremains uniform, and its initial radius is R0. Suppose further that the transversecomponents of the electrons’ momenta stay uniformly distributed over a circle in themomentum space , and the initial radius of this circle is P0.

a. If the focusing system reduces the beam radius from R0 to R1, how does thedistribution of transverse momenta change?

b. Part (a) is highly idealized. Suppose, instead, that the electron beam is bunched,and each electron bunch moves non- relativistically through the focusing system.Suppose further that Coulomb self-forces affect the beam, and that the focusingsystem imparts an external magnetic field Bext. Write down the Vlasov-Poissonkinetic equation for the distribution function of electrons in the six-dimensionalphase space of a single electron.

c. The Vlasov-Poisson equations are, in general, notoriously difficult to solve.Why?

d. Suppose, now, that the Hamiltonian for the beam bunch is independent of timeand that the distribution function is a function only of the Hamiltonian (again,with regards to the six-dimensional phase space of a single electron). Is thebeam in equilibrium? Explain your answer.

2. (Ito) The Joule coefficient may be written η ≡∂T∂v

u

=1cv

P −Tβκ

, and the Joule-

Thomson coefficient may be written µ ≡∂T∂P

h

=v

cP

Tβ −1( ). Here u is the internal

energy, β the expansivity, and κ the isothermal compressibility. Using these twocoefficients,

a. Find η and µ for a van der Waals gas andb. Show that both are zero for an ideal gas.

3. (Ito) Consider a two-level system with an energy 2ε separating upper and lowerstates. Assume that the energy splitting is the result of an external magnetic field B.

Given that the total magnetic energy is UB = −NεtanhεkT

, show that the associated

heat capacity is CB = Nk2εkT

2 e2ε kT

e2ε kT + 1( )2 .

4. (Benbow) Suppose you have large, long-focal length achromatic lens to use as theobjective of an astronomical telescope. We are told in geometrical optics that ifparallel rays are incident on a lens, the rays will converge to a point at the focal length

Page 66: Ph. D. Qualifying Exam Jan. 2017 Modern and Statistical Physics

of the lens.

If that is so, how can we expect to obtain an image of the moon (certainly not a pointsource) if we place a screen at the focal point of the lens? Explain, using words, notequations.

5. (Brown) Although a photon has no rest mass, it nevertheless interacts with electronsas though it has the inertial mass

mp p

c= =

υwhere the velocity of the photon is υ = c .

a. Compute the photon mass (in units of eV) for photons of wavelengths 5000 Α°

(visible region) and 1.0 Α° (x-ray region). Compare to the mass of an electron(which is 0.511 MeV). (Note: 154136 10 eV. sech −= × ⋅ ).

b. When we drop a stone of mass m from a height H near the earth’s surface, thegravitational pull of the earth accelerates it as it falls and the stone gains theenergy mgH on the way to the ground. A photon of frequency ν that falls in agravitational field gains energy, just as a stone does. Using the photon massrelation, determine the new frequency, ′ν , of the fallen photon, and thus thefrequency shift, ν ν− ′ , suffered by the photon.

c. A passenger in an airplane flying at 20,000 feet aims a laser beam of red light(λ = 7000 Α° ) towards the ground. What is the shift, λ λ′− , in the laser beamwavelength an observer on the ground would measure?

6. (Hedin) Consider the two reactions of an elastic scatter of a neutrino off an electron atrest for muon-type and electron-type neutrinos:

a) νµ + e à νµ + eb) νe + e à νe + e

a. Which will have the larger cross section for Eν = 1 GeV?b. Explain (best if you show the first order Feynman diagrams).c. For Eν =1 GeV, what is the maximum angle a scattered electron will have with

respect to the incoming neutrino?

Page 67: Ph. D. Qualifying Exam Jan. 2017 Modern and Statistical Physics

Modern Physics: Pick 4 of 6 1) The potential energy of gas molecules in a certain central field depends on the distance r from the field’s center as ( ) 2U r rα= where α is a positive constant. The gas temperature is T, the concentration of molecules at the center of the filed is n0.

a) Find the number of molecules dN, located at the distances between r and r+dr from the center of the field.

b) Find the most probable distance separating the molecules from the center of the field.

c) How many times will the concentration of molecules in the center of the field will change if the temperature decreases by η (Tnew = η × T). (Hint, how does dN/N, that is, the fraction of molecules located in spherical layer between r and r+dr, behave at the center)

d) Find the number of molecules dN, whose potential energy lies within the interval from U to U+dU.

FORMULA SHEET for PROBLEM 1

2

0

, 021 , 12

, 241 , 32

n x

n

nx e dx

n

n

π

π

∞−

= =

= = =

∫ 0

1, 0

1,2 21, 12, 2

n x

n

nx e dxnn

π∞−

= == = =

Page 68: Ph. D. Qualifying Exam Jan. 2017 Modern and Statistical Physics

2) Consider a particle of mass m undergoing Brownian motion in one dimension. The particle is under the influence of a viscous friction force –mβv, an oscillatory driving force –masin(ωt) and a random force mA(t). Its equations of motion are

( ) ( ), sin .x v v v a t A tβ ω= + = − +

a) Suppose ( ) 0A t = and ( )( ) ( )A t A tτ αδ τ= − , where α is a constant and δ(t) is the Dirac delta function. Suppose the initial conditions are q(0) =q0, v(0) = α/ω. Find the mean speed ( )v t . Note:

( )2 2

0

sin( ) cos( )sin( )

tt e t td e

ββτ β ω ω ω ω

τ ωτβ ω

− +=

+∫

b) Evaluate (by whatever means you chose) ( )v t in the limit β 0, and provide a physical interpretation of your result. 3) The density of states for a free particle of momentum p confined within a box of volume Ω is given by

2

32dn pdp π

Ω=

a) Calculate the density of states per unit energy for a non-relativistic electron and for a massless neutrino. Assume each is confined within a box of volume Ω but otherwise free of interactions. b) Assume that the transition probability for beta decay is dominated by the density of states term. Take the electron to be non-relativistic and the neutrino massless. In terms of the total decay energy, Etot, calculate the most likely energy for the emitted beta particle? 4) An energetic proton strikes a proton at rest. A K+ is produced. Write down a reaction for producing a K+ showing all the final state particles. What is the minimum kinetic energy for the incoming proton to produce this final state (express in terms of the masses of the particles)? 5) An interstellar proton interacts with the 3 degree cosmic microwave background (CMB) and produces an electron-positron pair via p+photon -> p+electron+positron. Assume the CMB is monoenergtic with E=kT= .002 eV. What is the minimum proton energy to produce this reaction? What is the electron energy at this threshold proton energy? The proton mass is 938 MeV/c2 and the electron mass is 0.5 MeV/c2.

Page 69: Ph. D. Qualifying Exam Jan. 2017 Modern and Statistical Physics

6)

A Michaelson interferometer has two arms, the first of length L and the second of length L+d. The interferometer is illuminated by a polychromatic source of light with a frequency distribution given by

( ) ( )22 20 0( ) / 2 exp / 2I Iν πσ ν ν σ = − −

a) Describe qualitatively how you expect the interference pattern for the case with polydisperse light to differ from the case where the interferometer is illuminated with a single frequency of light. b) Calculate the intensity of light observed as a function of the arm offset d. Does this result confirm your prediction from part a? You may want to use the following integral.

( )

( ) ( )2

2 222

2 22 cos 1 cos 22

x ykae kx dx ky e σπσ−∞ −

−∞

= + ∫

Polychromatic Light Source

Detector

Mirror

Mirror

Half Silvered Mirror

L L+d

Page 70: Ph. D. Qualifying Exam Jan. 2017 Modern and Statistical Physics

Statistical Mechanics: Pick 2 of 3. February,2005

1. Consider a large, spherical, self-gravitating system of identical stars for which thedistribution function in the phase space of a single star is

f (r,v) = CIH o - H (', O)]*'''t- an-3t2:c.lv?)-I,, | - ro.vr!,,L \/ 2 I 2

=o for Y <!r'2

Here Y = V(t) = Ho-Q(r) denotes a relative potential, with O(r)being the actualpotential, and ydenotes speed- Using Poisson's equation V'?@(r) = 4trGp(r) , find:

a) The density distribution function fr) conesponding to the index n = 5.

b) Are the stellar orbits in this system chaotic? Why or why not? Clearly explain yourreasoning.

2. T\e equation for the distribution of free electrons in a metal in the vicinity of absolutezero is

3/. ,l2m/2d"=:FJEatMaking use of this equation, find at T=0K

. a) The velocity distribution of free electrons.

b) The ratio of the mean velocity of free electrons to their maximum velocity-

c) What is the functional dependence of the Ferrni Energy on the density of electrons?

3. The distribution functions for identical particles (indistinguishable or distinguishable)can be represented by the equation:

Nr- I81

'Gt-r)'tr * oWhere a = 1 for Fermi-Dirac statistics, a= -1 for Bose-Einstein statistics and a=0 forMaxwell-B oltzmann statistics.

a) Sketch and compare N , I g, us. (e, - a) t nf for all three distriburions.b) For the Fermi-Dirac distribution, sketch N , I g, vs. e, for ?"= 0 and I slightly greaterthan zero.

Page 71: Ph. D. Qualifying Exam Jan. 2017 Modern and Statistical Physics

c) Show that for a system of a large number, N, of bosons at very low temperature (such

that they are all in nondegenerate lowest energy state t= 0), the chemical potential varieswith temperature according to:

p-+-kTlNasI-+0.

Modern Physics: Pick 2 out of 4

1. Consider an assembly of identical two state atoms in equilibrium with a radiationfield. The two states of the atom are labeled 1 and2 and their energy difference is hv-

state 2 has the higher enersy. Lnt p(v) =,yj\)denote the energv densitv

of the radiation field per unit frequency interval. The probability for stimulated emissionfrom state 2 to I and from state 1 to 2 are given by

W|r= Br,rP(v)

Wrt.r-- Br,rP(v)

The probability of spontaneous emission from state 2 is given by, W),t-- A.

a) What is the ratio N | / N z of the number of atoms in state 1, to the number of atoms in

state 2.b) Show thatB2l = Br,z and use this to find the rutio A/B'c) Explain briefly why it is necessary to have a non-thermal population distributionbetween two states in order to achieve laser action.

2. A pion-muon atom (that is the two particles are bound together) can be forrned in KLdecay via KL ) pion-muon atom + neutrino.

a) What are the energy levels of the pion-muon atom relative to the hydrogen atom?

b) What is the lifetime of the pion-muon atom in terms of the pion and muon lifetimes?

c) If the KL is at rest when it decays, what are the momentum and kinetic energy of thepi-mu atom?

Please state all answers in terms of the masses and lifetimes of the particles involved(pion, muon, kaon, proton, or electron).

3. Find the energy Q, expressed in terms of the masses of the atoms Mp, M.' and the

electron m", liberated in B- and B+ decays, and in K-electron-capture if the masses of theparent atom is Mn, the daughter atom M6, and an electron me are known. (Assumeelectron binding energies are negligible)-

Page 72: Ph. D. Qualifying Exam Jan. 2017 Modern and Statistical Physics

4. If two waves are to interfere, stipulations are that they must be traveling in the samedirection in the same region of space, and that their oscillatoryplanes be parallel. ff thislast condition is not met, something else will occur.

a. SupposeEx = Eoicos(kz- rn)E, = Eoicos(kz-m)

What happens here? What do you see if the wave is traveling toward you?b. Suppose

Et : Eolcos(kz- a)Ev = Eolsrntkz- ax1

'

Whathappens now? What do you see if the wave is traveling toward you?

Page 73: Ph. D. Qualifying Exam Jan. 2017 Modern and Statistical Physics

2.

J.

Ph.D. Qualifying Exam.statistical Mechanics, Thermodynamics and Mo dem Physics.

September 2004

Part L Statistical Mechanics Thermodynamics. @ick 2 out of 3) If you answer all thrreenote that only the first two will be graded.

1. A Carnot engine is operated between two heat reseryoirs at temperatures of 400 K to 300 K.(a) If the engine receives 1200 kilocalories from the reservoir at400K in each cycle,

how much heat does it reject to the reservoir at 300 K?(b) If the engine is operated as a refrigerator (i.e., in reverse) and receives 1200

kilocalories from the reservoir at 300 K, how much heat does it deliver to thereservoir at 400 K?

(c) How much work is done by the engine in each case?(d) What is the efiiciency of the engine in (a) and the coefficient of perfonnance in (bX

An ideal monatomic gas consists ofN atoms in a volume V. The gas is allowed to expendisothermally to fill a volume 2v. showthatthe enhopy change is-Nkln2.

A mercury atom moves in a cubical box whose edge is I m\ong. Its kinetic energJ is equalto the average kinetic energy of an atom of an ideal gas at 1000K. If the quantum numbersn", r7y, and n" are all equal to z, calculafe z. Note that the atomic mass of mercury is 200.6g/mol (hint calculate the mass of an individual atom).

Possible useful informationBoltzmannconstant, k: 1.38 x 10 -23 JK:8.617 x 10-5 eV/KStefan-Boltzmann constan! o =A[o=Ta :5.670x l0 JK4m-2s-r' l1h3ct-Planck's constant, h:6.62 x 10-3a JsSpeed of lighg c:2.99792458 x 108 ms-rR: 8.34 x 103 J kmole-r K-rdU: TdS - PdVH:U+PVF : U.TSG: F+PV

Values of constan8 : h-6.63x10-a.r.s, ft =1.38x1 0-23 J / K ,N e = 6.022x104 particles per mole.

Page 74: Ph. D. Qualifying Exam Jan. 2017 Modern and Statistical Physics

Part II. Modern Physics and Optics (Pick 2 out of 4)

1.. Assume that the mu-neutrino vrandthe tau neutrino y.are composed of a mixture of two mass

eigenstates vrandv, , with masses ml andm2.The mixing ratio is given by:

f",) _fcos(o) -sin(e)[v,\\",/ - \sin(o) cos(o) /\v,/

In free space, the states v1 andv2 ovolve according to

(lv r(x, t) r\ _ _*.,, ^("-'""'o [v, (o) t )fl",(",t) ,)- " \"-*""p,(o) '/

a) Show tlrat the transition probability for a mu-neufino to turn into a tau neutrino isgiven by:

PAt* ")= sin2(29)sin'l(nr- nrlt nnf

b) Show that in the extreme relativistic limit this result becomes:

PQt-- t) = sin'(2o1"rr'l(*l 't)st1t \,, L o* l" Herq I is the distance taveled by the neutrino, and E is the total energy.

2. A proton is confined to a2D square well of size

F 5 F (10^-15 m) suchthatV:-50MeV for0<xcD

o<y<Dand V:0 everywhere else.

a- Howmany energy levels are there inthis well?b. What is the degeneracy of each energy level?c. What is the approximate energy of the lowest energJ state? (and at least outline how

one can improve estimating the energy).

mass proton: 938 MeV/c^2 Hbar*c :197 MeV*F

Page 75: Ph. D. Qualifying Exam Jan. 2017 Modern and Statistical Physics

3. An electron with p : 1 GeV/c strikes a proton at rest.Their masses are 0.51 Meylc^2 and 93g MeV/c"2

a. What is the ma:rimtrm momenfum transverse to the incoming elecfion's direction thatthe exiting proton can have?

b. what is the maximum total energy that the exiting proton can have?c. What energy would the electron need to have to mate a rho meson, with mass of 770

MeV/C2? That is e+F>sFprrho.

4. For the waveform faveling in a dense glassE(r,t) =Xt(+i.3il(u/*)"os[o.oa+rr x t06e +t.rtlnx ldsr]

a^ Determine the ildex ofrefraction of the glass.b. Detennine the wavelength ofthe light in air.c. Resolve the" Eo" into an amplifude times a unitvector.d. Determine which directionthe wave is propagating.

Page 76: Ph. D. Qualifying Exam Jan. 2017 Modern and Statistical Physics

Ph.D. Qualifying Exam.Statistical Mechanics, Thermodynamics and Modern Physics.

January 2044

Part L statistical Mechanics Thermodynamics. (Pick 2 out of 3) If you answer all threenote that only the first two will be graded.

1) Calculate the total electomagnetic energy inside an oven of volume 1 m3 heated to atemperature of 600 degrees Fahrenheit.

2) Anideal monatomic gas undergoes a reversible expansion from specific volume vr tospecific volume vz.

(a) Calculate the change in specific entropy As if the expansion is isobaric.(b) Calculate As if the process is isothermal.(c) Whioh is larger? By how much?

3) For N distinguishable coins the thermodynamic probability i, , =ffi, where

Nr is the number of heads andN-Nr ttre number of tails.(a) Assume thatN is large enough that Stirling's approximation (lnn!-nlnn -n) is

valid. Show that lno is ma:rimum forNr : N/2.(b) Show that @^o-eNro2.

Possible useful informationBoltzmann constant, k: 1.38 x 10 -23 J/K: 8.617 x 10-5 eV/KStefan-Boltzmann constan! o:5.670 x 104 JK4m-2s-1Planck's constanL h:6.62x 10-34 Js

Speed of l^ight, c:2.99792458 x 108 ms-lR: 8-.34 x 103J knole-r K-ldU: TdS - PdVH:U+PVF : U.TSG: F+PV

Page 77: Ph. D. Qualifying Exam Jan. 2017 Modern and Statistical Physics

Part II. Modern Physics and Optics (Pick 2 out of 4)

1) A proton is confined to a 2D square well of size D: 5 F (10"t m) such that

V: -50 MeVandv:0

for0<x(D,0<y<D,everywhere else.

a- How many enerry levels are there in this well?b. What is the degeneracy of each energy level?c. What is the approximate energy of the lowest energy state? Give both the zeroth

and first order approximation.

Mass proton:938 MeV/c2 h*c:197 MeV*F

2) Consider a neufon star of mass M and radius R. Assume T: 0 K. Ignoringgravitational energy and assuming a simple Fermi gas model, what is the ma:<imumenergy a neutron can have? What is the average energy of the neutrons? Do notcalculate actual values but leave in the form using the mass of neufion, mo, and otherconstants.

3) A photon from the cosmic microwave backgfound with energy 1.2x10-3 eV(corresponding to a wavelength of I mm) collides head-on with a relativistic electronhaving kinetiCenerry 1 Ge\l The photon is backscattered i.e., scattered by 180o,

after which the electron continues to move in its initial direction. Find the energy ofthe photon after tlre collision. What is the wavelength of the backscattered photon?

4) Consider a plane electromagnetic wave incident in the normal direction on a systemof nnarrowslits each separated by a distance d. Derive a formula for the intensity ofthe soattering as a function of angle and wavelength in the Fraunhofferapproximation.

Page 78: Ph. D. Qualifying Exam Jan. 2017 Modern and Statistical Physics

Graduate O-*J?r"

Stat-Mech Pick2 of 3.

Problem 1:

Consider the Fokker-Planck equation for the distribution function f:

#*y, +q.ff = Q.(40*O(D.A),urhere 0 denotes a gradient with respect to velocity v. Take a simple model ofdynamical friction and dimlsion: = fy.and D = Dfr,, where F alrtdD ate constants

anA 1is the unit vector in the direction of y.

a- Describe the overall effect of the right-hand side on the evolution of the systernb. How is the coefFrcient Brelated to the force fluctuations a constituent particlee4periences?c. Deduce from the Fokker-Planck equation bw B afr D are related for a system inthermal equilibriurn

Problem2.

Consider the simplest model for desorption of atoms of mass m from a surface: one thatignored the tanslational degrees of freedonl and takes the possible energy states of theatom as -e (when adsorbed on the surfrce) and 0 when in the 'vacuum' at terperatureT.*, Use the canonical ensemble for Natoms to obtain an eqrression for the number ofatdms inthe vacuum.b. Wbat is the total energy ofthe system as a function oftemperature?

Problem 3.

Conduction electrons are confined in a metal at T= 300 K. The Fermi energy is 4 eV.What is the relative density of states N(E) for energy of 4.4 eV compared to 3.6 eV?What is the relative probability for anelectronto have enErgy of 4.4 eV compared to 3.6eV?

Page 79: Ph. D. Qualifying Exam Jan. 2017 Modern and Statistical Physics

F'- il 't c'6i

Modem/Optics Pick 2 of 4

Problem 4.

Derive Malus's law: Malus's law relates the intensityof light passing tbrotr$ttwo idealpolarizers with their polarizing directions at an angle A with each other. Polarizers acton the electric field vector (amplitude) of the light wave traversing thern An idealplafiznr passes 50% ofpurely unpolarized light incident on its surface, the remainderbeing now polarized parallel to the plarizng directionNow, armed with Malus's law, consider two ideal polarizers arranged so that a beam oflight passing through both finds the second one at an angle of 45'to the fust. Then a 3'dideal polarizer is inserted between them and rotated at angular frequency ar, Derive anexpression for the intensity ofthe light emitted from the system as a function oftime.

Problem 5.

Two neutrinos leave a supenrcva explosion at the same time. They arrive at earth adistance L away separated in time by At. The first neutrino to arrive is found to baveenergy Et andthe second energy Ez. Findthe mass ofthe neutrino.

Problem 6.

The disintegration constant )" of aradioactive nucleus is defined as the fraction of nucleithat decay per unit time. Let N(t) bethe number of nuclei present at time /.a. Derive the decay law, N(t): N(0)exp(-tr t)b. What is the relation between half-life Tn afr ,L2c. In an excitation experiment, a parent nucleusp is produced at arate.R, and decayswith a disintegration constant 2 p,to a daughter nucleus 4 whose disintegration constantsis La. At H, the number ofparent nuclei isifi0):N6oand the number of daughternuclei is ,va(0):0. Find nrlO and ila(Q for t>0, and show that after a long time, anequilibrium is attained.

Problem 7.

An electron is in the spin state

a. Determine the normal ^rr"^'"1"!r!^i)ob. Find the expectation values of S*, Sr, and Sr.

Page 80: Ph. D. Qualifying Exam Jan. 2017 Modern and Statistical Physics

NIU Ph.D./Master qualifier examination 2003 SpringStatistical Physics and Modern Physics

Solve 4 problems. Choose 2 problems from I, II and III, and choose 1 problemfrom V and VI, choose 1 problem from IV and VII.

I Consider a nonrelativistic free particle in a cubical container of edge length L and volume Zr3

-L

a Each quantum state r of this particle has a corresponding kinetic energy e,' which dependson Z What is t,(V)?

b Find the contribution to the gas pressure p, : -(dqldV) of aparticle in this state in terms^c ^ ^-) f/vL crdll(J v .

c Use this result to show that the mean pressure p of any ideal gas of weakly interacting

particles is always related to its mean total kinetic energy E AV V =1rlr, irrespective

of whether the gas obeys classical, Fermi-Dirac, or Bose-trinstein statistics.

II Consider a system of two atoms, each having only 3 quantum states of energies 0, e and 2e .

The system is n contact with aheatreservoir at temperature T. Write down the partitionfunction Z far the system if the particles obey

A. Classical statistics and are distinguishable.B. Classical statistics and are indistinguishable.C. Fermi-Dirac statistics.D. Bose-Einstein statistics.

lll A certain gas obeys the equation oistate P(v - b) = RT" where r? is the universal gas

constant, andb is a constant suchthat 0 <b <Vfot allV.

a Derive an expression for the work obtained from a reversible isothermal expression ofone mole of this gas from an initial volume Vito aftnalvohsne V7.

b If the gas were ideal, would the same process produce more or less work?

IVFindthethresholdenergyfortheprocess,y+p-uo*p,inwhichasingle'zrmesonis. produced when a photon strikes a proton at rest. The rest energy of the pion is 135 MeV and

the nroton 938 MeV.

V Take an ideal monatomic gas (y: 5/3) around the Carnot cycle, wherc point I atthebeginning of the adiabatic compression has pressure Pr : Pa (atmospheric presswe), volumeVt:13liters, andtemperatureTt:300 K. Point 3 has pressureP::2Po andvolume V3:26liters. Calculate the values of volume and pressure at all four points of the Carnot cycle.

Page 81: Ph. D. Qualifying Exam Jan. 2017 Modern and Statistical Physics

W An ideal gas initially at temperature, Ti, pressure, Pi and volume, V has its pressure reducedto a final value Pi< Pi via one of the following processes: (1) isochoric; (2) isothermal; (3)adiabatic.

a Sketch each process schematically on a P - V diagram.

b In which process is he work done on the gas zero?

c In which process is the work done on the gas greatest?

d Show that the ratio the absolute magnitudes of the heat transfers, Qi*t1,.r".-u1 to Qiro"1,o6" isgiven by

e In which process is the change in internal energy of the gas the greatest? Explain yourreasoning.

VII The k-alpha radiation from copper tZ19) occurs via the following process- An incidenthigh energy electron removes an electron from the n:1 orbital and subsequently an electronfrom the n:2 orbital falls down to the n:l orbital releasing an x-ray.

a Ignoring electron-electron interactions calculate, in units of the Rydberg constant R, theenergy of the Cu k-alpha x-ray.

b More realistically, the second 1S electron will shield the nuclear charge. Assumingperfect shielding, calculate a better estimate of the Cu k-alpha energy.

c Discuss, in qualitative terms, how inter-electron interactions might modifu the Cu k-alphaenergy, beyond just the perfect shielding approximation.

d The energy of the Cu k-alpha x-ray is slightly modified by the electron's spin-orbitinteraction. This leads to a splitting of the k-alpha line into a k-alpa-l and k-alpha-2.

i List all possible values of total (orbital + spin) angular momentum for the n:1 andn:2 states.

ii List all transitions from states with n:2 to states with n:l that are permitted by theselection rules for atomic transitions.

iii Explain the observ'ed ratio of 2:1 for the Cu k-alpha-l to k-alpha-2 fluorescenceyields.

leu,,*^^l= *e l,,lg)llril=4Eilt'tp ttlYisochnncl 'ul* ll \'/ ll